What is Exercise: Definition and 576 Discussions

Exercise is any bodily activity that enhances or maintains physical fitness and overall health and wellness.It is performed for various reasons, to aid growth and improve strength, preventing aging, developing muscles and the cardiovascular system, honing athletic skills, weight loss or maintenance, improving health and also for enjoyment. Many individuals choose to exercise outdoors where they can congregate in groups, socialize, and enhance well-being.In terms of health benefits, the amount of recommended exercise depends upon the goal, the type of exercise, and the age of the person. Even doing a small amount of exercise is healthier than doing none.

View More On Wikipedia.org
  1. G

    Electric field and electric potential exercise

    a) \vec{F}=\vec{E}\cdot q \phi =\oint \vec{E}d\vec{S}=\oint \vec{E}d\vec{S}=\underbrace{\oint \vec{E}d\vec{S}}_{\textrm{FACES } \perp}+\underbrace{\oint \vec{E}d\vec{S}}_{\textrm{FACES } \parallel}=0+\oint EdS\cdot \underbrace{\cos 0}_1= E2S \dfrac{Q_{enc}}{\varepsilon_0}=\phi \left...
  2. yam1244

    Solving this exercise in mechanics -- Tipping over this rectangular object

    A body is placed on a surface with friction Force is applied to the right at the upper end of the body What is the condition that will cause the body to roll over?
  3. jaumzaum

    Resistance Association - How to solve this exercise in the proper way?

    This was an exercise from the 2011 admission test of an university from Brazil: 40 identical generators with electromotive force 1,5V and resistance 0,25 Ohms and one resistor with resistance 2,5 Ohm are associated in such a way that the 2.5- resistor dissipates the greatest energy per second...
  4. T

    I Exploring Leverage & Force in Exercise Science

    Hi all, not exactly a physicist however I am finding that the expertise of those in this field, would be massively appreciated in the field I am currently delving into - exercise science. Specifically an understanding of engineering physics. As described above, in short I am trying to work out...
  5. jamalkoiyess

    Other Exercise book between underg and grad physics

    Hello, I am in my last year of undergrad and wanted a good book of diverse exercises to serve both as a memory refresher on my physics and as a fun pass time for this summer. Altho my goal is something that can span most of undergrad physics, I am not looking for anything easy and would enjoy...
  6. P

    Wind-Induced Bending Stress on Chimney with Rectangular Cross-Section

    A chimney has a rectangular cross-section with external dimensions 800 × 600 mm and wall thickness 150 mm. It is 5 m high. The density of the material is 2000 kg/m3. Assuming that the material is elastic, calculate the maximum uniform wind-pressure loading (N/m2) that...
  7. Svein

    Design a Ringbuffer for Thread Communication

    Design dokument for ”Ringbuffer” The basic design object is Ringbuffer: Object { private char buf[]; int buf_size, put_on, take_off; bool empty, full; int init(int size); int next(int ix); public int put(char ch); int get(void); } Additional design info The design goal...
  8. Stephen Bulking

    Electromagnetic Induction exercise, is something wrong with these answers?

    1. |ε|=dΦ/dt B=μ0I×cos(0-cos180)/4πr =μ0I/2πr dΦ=BdS=μ0I×(vdtxdr)/2πr ΔΦ=(μ0I×(vdt)/2π)x∫dr/r=(μ0I×(vdt)/2π)xln(b/a) *the intergral goes from a to b |ε|=dΦ/dt=(μ0I×v×ln(b/a)/2π) i=ε/R=(μ0I×v×ln(b/a)/2πR) 2. dI/dt=-50 B=μ0NI/L ε=-dΦ/dt=-BdS/dt (*) =-NBS/dt (wait what?)...
  9. Hajarmq

    Rotation and spring force exercise

    Summary:: Calculating the inclination angle A stick is on two springs with spring constants D1=500N/m and D2=300N/m. Consider the stick is without mass and can rotate around the point E, which is distant from spring 1 with 0,1m and from spring 2 with 0,8m. A force F=100N pulls the stick up...
  10. jedimath

    Exercise about Constant Motion

    NOTE: Sorry for my english. I use Google Translate! Comparing the performance with a friend of mine who is passionate about physics (and he is studying it by himself) we came to the same conclusion. In other words, we have calculated the time taken by both riders to reach the finish line. From...
  11. E

    Engineering Input-referred thermal noise voltage (Razavi exercise)

    So this is a problem from the legendary Razavi book "Design of Analog CMOS Integrated Circuits" (Specifically, it is the problem 7.7 (c)). I got the solutions online but when I try to calculate the noise expression myself, I don't arrive at the same result. Lambda is assumed zero for neglecting...
  12. JTorn

    I with an exercise about Energy and Schwarzschild Black Holes

    The thing is that this is an exercise that I have to show my teacher but I don´t know how to get the answer.The exercise says: "A body of mass m moving in the Keplerian field V = −M/r (in G = 1 units) has a total conserved energy, Etot = 1 /2( m r˙^2 + r ^2ϕ˙ ^2 )− mM/r. Show that the...
  13. C

    MHB Topology Munkres Chapter 1 exercise 2 e- Set theory

    Dear Everyone I am having some difficulties on exercise 2e from Topology 2nd ed by J. Munkres . Here are the directions: determine which of the following states are true for all sets A, B, C, and D. If a double implication fails, determine whether one or the other one of the possible...
  14. C

    MHB Topology Munkres Chapter 1 exercise 2 b and c- Set theory equivalent statements

    Dear Every one, I am having some difficulties on exercise 2b and 2c from Topology 2nd ed by J. Munkres . Here are the directions: determine which of the following states are true for all sets $A$, $B$, $C$, and $D$. If a double implication fails, determine whether one or the other one of the...
  15. Math Amateur

    MHB Open Balls in a Normed Vector Space .... Carothers, Exercise 32

    I am reading N. L. Carothers' book: "Real Analysis". ... ... I am focused on Chapter 3: Metrics and Norms ... ... I need help Exercise 32 on page 46 ... ... Exercise 32 reads as follows: I have not been able to make much progress ... We have ...B_r(x) = \{ y \in M \ : \ d(x, y) \lt r \}...
  16. dude2

    A Aharonov - Bohm effect exercise

    Does anyone know the answers to this, or can hopefully guide me to a text that will help me solve this aharonov-bohm problem? Here is the given: Particles (of mass m, and charge q), are driven through two slits that have distance d between them, in a screen that is far away (L>>d) from the...
  17. babaliaris

    Thermodynamics: I can't solve this simple entropy exercise....

    The text says: "Steel bullet of 25kg with a Temperature of 400 Celsius, is being dropped on the bottom of an oil liquid of 100kg at a temperature of 100 Celsius. The system is isolated. Calculate a) The change of entropy of the bullet, b) the change of entropy of the oil, c) the total change of...
  18. B

    Kinematics- Exercise: Driving a car on a Motorway (1eq, 2variables)

    The car in front of us (2.car): ## s_2=v_0t ## The car that is accelerating (1.car): ## s_1=x+2l+s_2= \frac{1}{2} at^2 ## Now, if we substitute the equations, we get ## x+ 2l+ v_0t= \frac{1}{2} at^2 ##. I have now 1equation with 2variables (a, t)- any suggestions on how to continue?
  19. MaxR2018

    Digital signal exercise (Manolakis)

    Hello people, I hope someone can help me with this exercise. This exercise is taken from the book "Applied Digital Signal Processing: Theory and Practice" by manolakis and Ingle. I don't understand what is being asked. The only thing I understand is that I have to use MATLAB necessarily. Someone...
  20. Math Amateur

    MHB Infinite Series .... Sohrab Exercise 2.3.10 (1) .... ....

    I am reading Houshang H. Sohrab's book: "Basic Real Analysis" (Second Edition). I am focused on Chapter 2: Sequences and Series of Real Numbers ... ... I need help In order to formulate a rigorous proof to the proposition stated in Exercise 2.3.10 (1) ... ... Exercise 2.3.10 (1) reads as...
  21. Nekomimi

    An exercise about rationalizing denominators

    Express the following as a fraction with rational denominator: $$\frac{5^{\frac{1}{3}}}{5^{\frac{5}{3}}}$$ If I try to start by multiplicating both the numerator and denominator by ##5^{-\frac{2}{3}}##, I get: $$\begin{align} \nonumber \frac{5^{\frac{1}{3}}}{5^{\frac{5}{3}}} & =...
  22. A

    Solving Exercise: Find Thickness of Rubber Insulation for Copper Wire

    Good day all, I tried to solve the following exercise, I would be glad to get feedback for my attempt ( indeed I was puzzled by the numerical solution I got just need to check it for any mistake: A copper wire , 2mm in diameter and 100 m in length has an electrical resistance of 0,5 mΩ/m, the...
  23. O

    What is the period of the second planet in this exercise?

    Homework Statement In a distant galaxy, a planet orbits its sun at a distance of m with a period of 108 s. A second planet orbits the same sun at a distance of m. What is the period of the second planet? Select one: a. s b. s c. s d. s e. Homework Equations T^2=constant * r^3 The...
  24. Alex Langevub

    An exercise with the third isomorphism theorem in group theory

    Homework Statement Let ##G## be a group. Let ##H \triangleleft G## and ##K \leq G## such that ##H\subseteq K##. a) Show that ##K\triangleleft G## iff ##K/H \triangleleft G/H## b) Suppose that ##K/H \triangleleft G/H##. Show that ##(G/H)/(K/H) \simeq G/K## Homework Equations The three...
  25. Math Amateur

    MHB 1-Forms .... Bachman, Exercise 3.2, Part 2 ....

    I am reading David Bachman's book: "A Geometric Approach to Differential Forms" (Second Edition) ... I need some help with Exercise 3.2, Part 2 ... Exercise 3.2, Part 2 ... reads as follows:Bachman gives he answer to Exercise 3.2, Part 2 as dy = -4dx ... ... but ... I cannot understand how...
  26. W

    Engineering Grounding Op-Amp circuit exercise

    Homework Statement 2. Homework Equations 3. The Attempt at a Solution For the first set of questions: I've worked through to part 6), at which is I encountered my first problem. I'm not entirely sure what the question is asking. Is it as if there would be a capacitor between...
  27. C

    Bragg diffraction form an “inclined” crystal plane

    Homework Statement In picture, first-order reflection from the reflection planes shown occurs when an x-ray beam of wavelength ##0.260 nm## makes an angle ##\theta=63.8°## with the top face of the crystal. What is the unit cell size ##a_0##? Homework Equations Bragg law $$d=\frac{ n...
  28. Alex Petrosyan

    Studying Are there any good exercise books for Quantum Mechanics?

    hi, I’ve been walking through posts on this forum, and found plenty of book recommendations for QM. My problem is, I haven’t had enough practice in a long while. I could go over Solved Problems in e.g. Galitsky, or Gasiorowitz, but they seem to be a little far from what I want to test...
  29. X

    Help with a simple exercise in astrophysics.

    <Moderator's note: Moved from a technical forum and thus no template.> So i began studying on my own for my astrophycis class that will start on 2 weeks and i have trouble solving an exercise. So the exercise goes as follows: given that mbol=-2.5log(∫fλdλ)+Cbol and mbol=-26.83 for sun. Show...
  30. Math Amateur

    MHB Help w/ Awodey Exercise 5, Ch. 1: Functor from Slice Category to Any Category

    I am reading Steve Awodey's book: Category Theory (Second Edition) and am focused on Chapter 1: Categories I need some help in order to make a meaningful start on Awodey Exercise 5, Chapter 1 Awodey Exercise 5, Chapter 1 reads as follows: I am unable to make a meaningful start on this...
  31. Math Amateur

    MHB Functor From Any Category to Preorder as a Category .... Awodey Exercise 8, Chapter 1 ....

    I am reading Steve Awodey's book: Category Theory (Second Edition) and am focused on Chapter 1: Categories I need some help in order to make a meaningful start on Awodey Exercise 8, Chapter 1 Awodey Exercise 8, Chapter 1 reads as follows: I am unable to make a meaningful start on this...
  32. G

    A transmission system exercise

    Homework Statement We have a transmission system (plate-pinion), with a 38-tooth chainring and a 14-tooth sprocket. The distance between the crank (between the chainring and the pedal) is 170 mm and the pedal is overloaded with 60 kg and pedalled at a speed of 70 min-1. Homework Equations P =...
  33. F

    Doubt on an exercise involving Torque

    Homework Statement In the following figure we have an homogeneous L-shaped bar that has weight 120N. It's articulated without friction at point A. It's vertical part has length 1m and its horizontal part has length 3m. Find the intensity of F that maintains the bar in equilibrium. 2 ...
  34. Pushoam

    Checking the linear independence of elements of 2 X 2 matrices

    Homework Statement Homework Equations 3. The Attempt at a Solution [/B] ## |3 \rangle = |1 \rangle - 2 ~ |2 \rangle ## So, they are not linearly independent. One way to find the coefficients is : ## |3 \rangle = a~ |1 \rangle +b~ |2 \rangle ## ...(1) And solve (1) to get the values of a...
  35. Math Amateur

    MHB Another Question on Torsion Elements .... D&F Section 10.1, Exercise 8 ....

    I am reading Dummit and Foote's book: "Abstract Algebra" (Third Edition) ... I am currently studying Chapter 10: Introduction to Module Theory ... ... I need some help with an Exercise 8(c) of Section 10.1 ... Exercise 8 of Section 10.1 reads as...
  36. Pushoam

    Functions forming a vector space

    Homework Statement 1.1.3 1) Do functions that vanish at the endpoints x=0 and L=0 form a vector space? 2) How about periodic functions? obeying f(0)=f(L) ? 3) How about functions that obey f(0)=4 ? If the functions do not qualify, list what go wrong.Homework Equations The Attempt at a...
  37. D

    Maximizing Brain Function: The Link Between Exercise and Cognitive Ability

    Is it a mainstream agreed fact that our brains can only grow or we became smart only if we have regular exercise? Meaning if you never exercise.. your brain can't be optimal and you would have difficulty understanding physics and math? What kind of exercise do you do? I heard it should be...
  38. M

    Low Air Pressure Training (exercise)

    Looking to do a experiment on the effects of air resistance training on the body. doing basic exercises such as jogging push ups. Maybe a couple of curls ;).got facility design herewondering what your thoughts and opinions on this would be.
  39. Math Amateur

    MHB R[X] is never a field .... Sharp, Exercise 1.29 .... ....

    I am reading R. Y. Sharp's book: "Steps in Commutative Algebra" Cambridge University Press (Second Edition) ... ... I am focused on Chapter 1: Commutative Rings and Subrings ... ... I need some help with Exercise 1.29 ... Exercise 1.29 reads as follows:I am somewhat unsure about how to go...
  40. Math Amateur

    R[X] is never a field .... Sharp, Exercise 1.29 .... ....

    Homework Statement I am reading R. Y. Sharp's book: "Steps in Commutative Algebra" Cambridge University Press (Second Edition) ... ... I am focused on Chapter 1: Commutative Rings and Subrings ... ... I need some help with Exercise 1.29 ... Exercise 1.29 reads as follows: Homework...
  41. P

    Frenkel/Smit: Exercise 10 - MD NVE Code, PBC

    Homework Statement I am working on a MD code (http://www.acmm.nl/molsim/frenkel_smit/Exercise_10/index.html) which uses the position Verlet algorithm to integrate the equations of motion and a velocity rescaling algorithm for initialization in periodic boundary conditions. I wondered why...
  42. alejandromeira

    Special Relativity Exercise (A.P. French)

    I just solved an exercise in the special relativity book of A.P. French. I would like you to tell me if the answer is correct. Yes I suposse is a very easy exersice. I hope you can help me, because I really like learn relativity and cosmology, but I have not had teachers of this, and I am...
  43. Justin Bishop

    Misc. Safe Pull-up Bar Span for 124" - Advice?

    Hello smart physicists! I'm hoping to run a span of 124" for a pull up bar, with a support in the middle at the 62" point (so 3 supports total). I'm using the Rogue P-5V Pull-Up Bar Brackets (https://www.roguefitness.com/rogue-p-5v-garage-pullup-system). The brackets call for a tube with a...
  44. nomadreid

    I Connes' non-commutative geometry: useful or just an exercise?

    I know about the construction of the algebra in which operators as in Hilbert spaces are developed from Connes' non-commutative geometry, but I don't find any references [besides further publications by Connes himself] which say that this has turned out to be useful in physics for more than a...
  45. J

    How Do Position and Momentum Operators Act in Quantum Mechanics?

    Yesterday, I was solving an exercise from Cohen-Tannoudji's book - Quantum Mechanics -, but then I got stuck on the second question that the exercise brings. I wonder if you guys could help me, and here is the exercise: "Using the relation <x|p> = (2πħ)-½ eipx/ħ, find the expressions <x|XP|ψ>...
  46. F

    Q factor of [exercise] battle ropes vs. elastic tubing?

    Hello. I'm providing a technical review of a new exercise device: the "inertia wave". . The inventor knows this device is fundamentally different from battle ropes. After playing with one for a few days, I agree. I think that the simplest way to explain this to civilians is through the...
  47. J

    Need help with mechanics exercise - Vertical motion of 3 balls

    Homework Statement So I've been stuck on this exercise for a few hours now, maybe you can help out: 3 balls meet at the same height hm. Ball 1 is accelerated straight up into the air for 1s from height 0. When the acceleration stops, ball 2 is launched straight up from height 0 with v2 =...
  48. jim mcnamara

    Medical Aging, exercise, and immunosenscence

    http://onlinelibrary.wiley.com/doi/10.1111/acel.12750/full The effect of lifelong exercise decreases the immune system's degradation with age. Lifelong cyclists were compared to more sedentary age-matched counterparts. Several markers for immune system activity level were found to be higher...
  49. H

    What is the Method for Finding Area Between Curves?

    Homework Statement Homework Equations The Attempt at a Solution I'm confused avout questions 2-3. The answers for 2-2 is 1 So the answer for 2-3 is $$\frac{1}{3}$$ But, how the area looks like? Because $$ x^2 $$ will be an open curve upside? There's no boundary for above side.
Back
Top